matheraum.de
Raum für Mathematik
Offene Informations- und Nachhilfegemeinschaft

Für Schüler, Studenten, Lehrer, Mathematik-Interessierte.
Hallo Gast!einloggen | registrieren ]
Startseite · Forum · Wissen · Kurse · Mitglieder · Team · Impressum
Forenbaum
^ Forenbaum
Status Mathe
  Status Schulmathe
    Status Primarstufe
    Status Mathe Klassen 5-7
    Status Mathe Klassen 8-10
    Status Oberstufenmathe
    Status Mathe-Wettbewerbe
    Status Sonstiges
  Status Hochschulmathe
    Status Uni-Analysis
    Status Uni-Lin. Algebra
    Status Algebra+Zahlentheo.
    Status Diskrete Mathematik
    Status Fachdidaktik
    Status Finanz+Versicherung
    Status Logik+Mengenlehre
    Status Numerik
    Status Uni-Stochastik
    Status Topologie+Geometrie
    Status Uni-Sonstiges
  Status Mathe-Vorkurse
    Status Organisatorisches
    Status Schule
    Status Universität
  Status Mathe-Software
    Status Derive
    Status DynaGeo
    Status FunkyPlot
    Status GeoGebra
    Status LaTeX
    Status Maple
    Status MathCad
    Status Mathematica
    Status Matlab
    Status Maxima
    Status MuPad
    Status Taschenrechner

Gezeigt werden alle Foren bis zur Tiefe 2

Navigation
 Startseite...
 Neuerdings beta neu
 Forum...
 vorwissen...
 vorkurse...
 Werkzeuge...
 Nachhilfevermittlung beta...
 Online-Spiele beta
 Suchen
 Verein...
 Impressum
Das Projekt
Server und Internetanbindung werden durch Spenden finanziert.
Organisiert wird das Projekt von unserem Koordinatorenteam.
Hunderte Mitglieder helfen ehrenamtlich in unseren moderierten Foren.
Anbieter der Seite ist der gemeinnützige Verein "Vorhilfe.de e.V.".
Partnerseiten
Dt. Schulen im Ausland: Mathe-Seiten:Weitere Fächer:

Open Source FunktionenplotterFunkyPlot: Kostenloser und quelloffener Funktionenplotter für Linux und andere Betriebssysteme
StartseiteMatheForenLineare Algebra - MatrizenRing auf Matrizen beweisen
Foren für weitere Studienfächer findest Du auf www.vorhilfe.de z.B. Astronomie • Medizin • Elektrotechnik • Maschinenbau • Bauingenieurwesen • Jura • Psychologie • Geowissenschaften
Forum "Lineare Algebra - Matrizen" - Ring auf Matrizen beweisen
Ring auf Matrizen beweisen < Matrizen < Lineare Algebra < Hochschule < Mathe < Vorhilfe
Ansicht: [ geschachtelt ] | ^ Forum "Lineare Algebra - Matrizen"  | ^^ Alle Foren  | ^ Forenbaum  | Materialien

Ring auf Matrizen beweisen: Frage (beantwortet)
Status: (Frage) beantwortet Status 
Datum: 03:00 So 01.11.2015
Autor: Hamd.44

Aufgabe
Betrachte die Menge [mm] \IR^{2x2} [/mm] := { [mm] {\pmat{ a & b \\ c & d } | a,b,c,d } \in \IR} [/mm] der 2x2 Matrizen mit reellen Einträgen. Definiere Operationen auf [mm] R^{2x2} [/mm] mittels
        [mm] \IR^{2x2} [/mm] x [mm] \IR^{2x2} \to \IR^{2x2} [/mm]
( [mm] \pmat{ a_{1} & a_{2} \\ a_{3} & a_{4} }, \pmat{ b_{1} & b_{2} \\ b_{3} & b_{4} }) \mapsto \pmat{ a_{1} & a_{2} \\ a_{3} & a_{4} } [/mm] * [mm] \pmat{ b_{1} & b_{2} \\ b_{3} & b_{4} } [/mm] := [mm] \pmat{ a_{1}b_{1} + a_{2}b_{3} & a_{1}b_{2} + a_{2}b_{4} \\ a_{3}b_{1} + a_{4}b_{3} & a_{3}b_{2} + a_{4}b_{4} } [/mm]
( [mm] \pmat{ a_{1} & a_{2} \\ a_{3} & a_{4} }, \pmat{ b_{1} & b_{2} \\ b_{3} & b_{4} }) \mapsto \pmat{ a_{1} & a_{2} \\ a_{3} & a_{4} } [/mm] + [mm] \pmat{ b_{1} & b_{2} \\ b_{3} & b_{4} } [/mm] := [mm] \pmat{ a_{1} + b_{1} & a_{2} + b_{2} \\ a_{3} + b_{3} & a_{4} + b_{4} } [/mm]

(a) Zeige, dass [mm] R^{2x2} [/mm] mit dieser Multiplikation und Addition ein Ring ist. Ist [mm] R^{2x2} [/mm] kommutativ?

(b) Betrachte die Teilmenge
c := { [mm] \pmat{a & -b \\ b & a} \in \IR^{2x2} [/mm] | a,b [mm] \in \IR [/mm] }
Zeige, dass C ein Unterring von [mm] R^{2x2} [/mm] ist. Ist C kommutativ?

Hey Leute,

ich bräuchte nochmals eure Hilfe, wäre nett wenn ihr mir wieder einen Schubes in die Richtung geben könntet.

Zu a)
Zunächst muss ich folgende 3 Sachen zeigen, damit es sich auch wirklich um einen Ring handelt.
R1) [mm] (\pmat{ a_{1} & a_{2} \\ a_{3} & a_{4} }, [/mm] +) ist eine ablege Gruppe
R2) Die Verknüpfung "*" ist assoziativ
R3) Es gelten die Distributivgesetze, d.h.
( [mm] \pmat{ a_{1} & a_{2} \\ a_{3} & a_{4} } [/mm] + [mm] \pmat{ b_{1} & b_{2} \\ b_{3} & b_{4} } [/mm] ) * [mm] \pmat{ c_{1} & c_{2} \\ c_{3} & c_{4} } [/mm] = [mm] \pmat{ a_{1} & a_{2} \\ a_{3} & a_{4} } [/mm] * [mm] \pmat{ c_{1} & c_{2} \\ c_{3} & c_{4} } [/mm] + [mm] \pmat{ b_{1} & b_{2} \\ b_{3} & b_{4} } [/mm] * [mm] \pmat{ c_{1} & c_{2} \\ c_{3} & c_{4} } [/mm]
und
[mm] \pmat{ a_{1} & a_{2} \\ a_{3} & a_{4} } [/mm] * ( [mm] \pmat{ b_{1} & b_{2} \\ b_{3} & b_{4} } [/mm] + [mm] \pmat{ c_{1} & c_{2} \\ c_{3} & c_{4} } [/mm] ) = [mm] \pmat{ a_{1} & a_{2} \\ a_{3} & a_{4} } [/mm] * [mm] \pmat{ b_{1} & b_{2} \\ b_{3} & b_{4} } [/mm] + [mm] \pmat{ a_{1} & a_{2} \\ a_{3} & a_{4} } [/mm] * [mm] \pmat{ c_{1} & c_{2} \\ c_{3} & c_{4} } [/mm]
-----------------------------------------------------------------------------------
Mein Lösungsvorschlag
R1) Da die Matrizenaddition komponenterweise über Körperelemente definiert ist, bildet sie automatisch eine abelsche Gruppe

R2) Zu beweisen ist, dass gilt:
M_(a) * ( M_(b) * M_(c) )= ( M_(a) * M_(b) ) * M_(c)
Ich würde dass nun per Definition zur Matrizenmultiplikation einfach zeigen dass die Aussagen Äquivalent zu einander sind.

R3) Äquivalent zu R2)
-------
Um die Kommutativität zu zeigen, fällt mir nichts ein außer zu zeigen, dass gilt:
M_(a) * M_(b) = M_(b) * M_(a)
und
M_(a) + M_(b) = M_(b) + M_(a)
Gibt es dazu vielleicht einen kürzeren Weg, sonst ist das ja nur unnötig viel Schreib-Arbeit... :S

Über eine Rückmeldung würde ich mich freuen.


Viele Grüße,
Hamd.44

        
Bezug
Ring auf Matrizen beweisen: Antwort
Status: (Antwort) fertig Status 
Datum: 07:59 So 01.11.2015
Autor: fred97


> Betrachte die Menge [mm]\IR^{2x2}[/mm]Eingabefehler: "{" und "}" müssen immer paarweise auftreten, es wurde aber ein Teil ohne Entsprechung gefunden (siehe rote Markierung)

:= { [mm]{\pmat{ a & b \\ c & d } | a,b,c,d } \in \IR}[/mm]

> der 2x2 Matrizen mit reellen Einträgen. Definiere
> Operationen auf [mm]R^{2x2}[/mm] mittels
>          [mm]\IR^{2x2}[/mm] x [mm]\IR^{2x2} \to \IR^{2x2}[/mm]
>  ( [mm]\pmat{ a_{1} & a_{2} \\ a_{3} & a_{4} }, \pmat{ b_{1} & b_{2} \\ b_{3} & b_{4} }) \mapsto \pmat{ a_{1} & a_{2} \\ a_{3} & a_{4} }[/mm]
> * [mm]\pmat{ b_{1} & b_{2} \\ b_{3} & b_{4} }[/mm] := [mm]\pmat{ a_{1}b_{1} + a_{2}b_{3} & a_{1}b_{2} + a_{2}b_{4} \\ a_{3}b_{1} + a_{4}b_{3} & a_{3}b_{2} + a_{4}b_{4} }[/mm]
>  
> ( [mm]\pmat{ a_{1} & a_{2} \\ a_{3} & a_{4} }, \pmat{ b_{1} & b_{2} \\ b_{3} & b_{4} }) \mapsto \pmat{ a_{1} & a_{2} \\ a_{3} & a_{4} }[/mm]
> + [mm]\pmat{ b_{1} & b_{2} \\ b_{3} & b_{4} }[/mm] := [mm]\pmat{ a_{1} + b_{1} & a_{2} + b_{2} \\ a_{3} + b_{3} & a_{4} + b_{4} }[/mm]
>  
> (a) Zeige, dass [mm]R^{2x2}[/mm] mit dieser Multiplikation und
> Addition ein Ring ist. Ist [mm]R^{2x2}[/mm]Eingabefehler: "{" und "}" müssen immer paarweise auftreten, es wurde aber ein Teil ohne Entsprechung gefunden (siehe rote Markierung)

kommutativ?

>  
> (b) Betrachte die Teilmenge
>  c := { [mm]\pmat{a & -b \\ b & a} \in \IR^{2x2}[/mm] | a,b [mm]\in \IR[/mm]Eingabefehler: "{" und "}" müssen immer paarweise auftreten, es wurde aber ein Teil ohne Entsprechung gefunden (siehe rote Markierung)


> }
>  Zeige, dass C ein Unterring von [mm]R^{2x2}[/mm] ist. Ist C
> kommutativ?
>  Hey Leute,
>  
> ich bräuchte nochmals eure Hilfe, wäre nett wenn ihr mir
> wieder einen Schubes in die Richtung geben könntet.
>  
> Zu a)
>  Zunächst muss ich folgende 3 Sachen zeigen, damit es sich
> auch wirklich um einen Ring handelt.
>  R1) [mm](\pmat{ a_{1} & a_{2} \\ a_{3} & a_{4} },[/mm] +) ist eine
> ablege Gruppe
>  R2) Die Verknüpfung "*" ist assoziativ
> R3) Es gelten die Distributivgesetze, d.h.
> ( [mm]\pmat{ a_{1} & a_{2} \\ a_{3} & a_{4} }[/mm] + [mm]\pmat{ b_{1} & b_{2} \\ b_{3} & b_{4} }[/mm]
> ) * [mm]\pmat{ c_{1} & c_{2} \\ c_{3} & c_{4} }[/mm] = [mm]\pmat{ a_{1} & a_{2} \\ a_{3} & a_{4} }[/mm]
> * [mm]\pmat{ c_{1} & c_{2} \\ c_{3} & c_{4} }[/mm] + [mm]\pmat{ b_{1} & b_{2} \\ b_{3} & b_{4} }[/mm]
> * [mm]\pmat{ c_{1} & c_{2} \\ c_{3} & c_{4} }[/mm]
>  und
> [mm]\pmat{ a_{1} & a_{2} \\ a_{3} & a_{4} }[/mm] * ( [mm]\pmat{ b_{1} & b_{2} \\ b_{3} & b_{4} }[/mm]
> + [mm]\pmat{ c_{1} & c_{2} \\ c_{3} & c_{4} }[/mm] ) = [mm]\pmat{ a_{1} & a_{2} \\ a_{3} & a_{4} }[/mm]
> * [mm]\pmat{ b_{1} & b_{2} \\ b_{3} & b_{4} }[/mm] + [mm]\pmat{ a_{1} & a_{2} \\ a_{3} & a_{4} }[/mm]
> * [mm]\pmat{ c_{1} & c_{2} \\ c_{3} & c_{4} }[/mm]
>  
> -----------------------------------------------------------------------------------
>  Mein Lösungsvorschlag
>  R1) Da die Matrizenaddition komponenterweise über
> Körperelemente definiert ist, bildet sie automatisch eine
> abelsche Gruppe

Das ist mir zu knapp.


>  
> R2) Zu beweisen ist, dass gilt:
>  M_(a) * ( M_(b) * M_(c) )= ( M_(a) * M_(b) ) * M_(c)
> Ich würde dass nun per Definition zur
> Matrizenmultiplikation einfach zeigen dass die Aussagen
> Äquivalent zu einander sind.

Du meinst obige Gleichheit ? Wenn ja, so kannst Du das so machen.


>  
> R3) Äquivalent zu R2)


Was meinst Du denn damit ???



>  -------
>  Um die Kommutativität zu zeigen, fällt mir nichts ein
> außer zu zeigen, dass gilt:
>  M_(a) * M_(b) = M_(b) * M_(a)


Das wird Dir nicht gelingen, denn die Matrixmultiplikation ist nicht kommutativ






> und
>  M_(a) + M_(b) = M_(b) + M_(a)

Das kannst Du zeigen



Fred

>  Gibt es dazu vielleicht einen kürzeren Weg, sonst ist das
> ja nur unnötig viel Schreib-Arbeit... :S
>  
> Über eine Rückmeldung würde ich mich freuen.
>  
>
> Viele Grüße,
>  Hamd.44


Bezug
                
Bezug
Ring auf Matrizen beweisen: Frage (beantwortet)
Status: (Frage) beantwortet Status 
Datum: 00:21 Mo 02.11.2015
Autor: Hamd.44

Hmmmm, ich habe nun R2) und R3) komplett gezeigt, jedoch bin ich bei R1) noch etwas unsicher. Ich soll zeigen dass (R, +) albesch bzw. kommutativ ist.
D.h.:
[mm] \pmat{ a_{1} & a_{2} \\ a_{3} & a_{4} } [/mm] + [mm] \pmat{ b_{1} & b_{2} \\ b_{3} & b_{4} } [/mm] = [mm] \pmat{ b_{1} & b_{2} \\ b_{3} & b_{4} } [/mm] + [mm] \pmat{ a_{1} & a_{2} \\ a_{3} & a_{4} } [/mm]
Stimmt das so?

Vg,
Hamd.44

Bezug
                        
Bezug
Ring auf Matrizen beweisen: Antwort
Status: (Antwort) fertig Status 
Datum: 05:31 Mo 02.11.2015
Autor: fred97


> Hmmmm, ich habe nun R2) und R3) komplett gezeigt, jedoch
> bin ich bei R1) noch etwas unsicher. Ich soll zeigen dass
> (R, +) albesch bzw. kommutativ ist.
>  D.h.:
>  [mm]\pmat{ a_{1} & a_{2} \\ a_{3} & a_{4} }[/mm] + [mm]\pmat{ b_{1} & b_{2} \\ b_{3} & b_{4} }[/mm]
> = [mm]\pmat{ b_{1} & b_{2} \\ b_{3} & b_{4} }[/mm] + [mm]\pmat{ a_{1} & a_{2} \\ a_{3} & a_{4} }[/mm]
>  
> Stimmt das so?

ja, das stimmt. warum ?

fred

>  
> Vg,
>  Hamd.44


Bezug
                                
Bezug
Ring auf Matrizen beweisen: Frage (überfällig)
Status: (Frage) überfällig Status 
Datum: 20:40 Mo 02.11.2015
Autor: Hamd.44

[mm] \pmat{ a_{1} & a_{2} \\ a_{3} & a_{4} } [/mm] + [mm] \pmat{ b_{1} & b_{2} \\ b_{3} & b_{4} } [/mm] = [mm] \pmat{ a_{1} + b_{1} & a_{2} + b_{2} \\ a_{3} + b_{3} & a_{4} + b_{4} } [/mm] = [mm] \pmat{ b_{1} + a_{1} & b_{2} + a_{2} \\ b_{3} + a_{3} & b_{4} + a_{4} } [/mm] = [mm] \pmat{ b_{1} & b_{2} \\ b_{3} & b_{4} } [/mm] + [mm] \pmat{ a_{1} & a_{2} \\ a_{3} & a_{4} } [/mm]

Das sollte so als Beweis für die abelsche Gruppe reichen, oder?

Hat jemand vielleicht noch einen Ansatz für b) Zur Untergruppe und wie ich zeigen kann, dass es sich um einen Unterring handelt. :S
Wüsste jetzt keinen wirklichen Ansatz.

Vg,
Hamd.44

Bezug
                                        
Bezug
Ring auf Matrizen beweisen: Fälligkeit abgelaufen
Status: (Mitteilung) Reaktion unnötig Status 
Datum: 21:20 Mi 04.11.2015
Autor: matux

$MATUXTEXT(ueberfaellige_frage)
Bezug
Ansicht: [ geschachtelt ] | ^ Forum "Lineare Algebra - Matrizen"  | ^^ Alle Foren  | ^ Forenbaum  | Materialien


^ Seitenanfang ^
www.matheraum.de
[ Startseite | Forum | Wissen | Kurse | Mitglieder | Team | Impressum ]